What is 3/4 divided by 1/2

Answers

Answer 1

Answer:

6/4 or 1 1/2

Step-by-step explanation:

You have to use the Keep Change Flip technique.

Keep 3/4.

Change the division symbol to a multiplication one

Flipi 1/2 to 2/1

and when you multiply the answer is 6/4 = 1 1/2


Related Questions

Which number completes the system of linear
inequalities represented by the graph?
y> 2x – 2 and x + 4y > _____

Answers

The system of linear inequalties that is represented by the graph is formed by the expressions y > 2 · x - 2 and x + 4 · y > - 12.

What system of inequalities generate the graph seen in the image?

Herein we have two inequalities generated by two linear functions whose form is described below:

f(x, y) > a (blue region)      (1)

g(x, y) > b (red region)      (2)

By direct comparison, we find that the limit of the red region is represented by the inequality y > 2 · x - 2. From the equation of the line we can derive an expression for the limit of the blue region:

Slope

m = [- 4 - (- 3)]/(4 - 0)

m = - 1/4

Intercept

b = y - m · x

b = - 3 - (- 1/4) · 0

b = - 3

Then, the inequation is represented by the linear equation:

y > - (1/4) · x - 3

4 · y > - x - 12

x + 4 · y > - 12

The system of linear inequalties that is represented by the graph is formed by the expressions y > 2 · x - 2 and x + 4 · y > - 12.

To learn more on inequalities: https://brainly.com/question/20383699

#SPJ1

Which number produces an irrational number when multiplied by 3/4

Answers

any irrational number would do, like pi or root(2)

A line with a slope of 3 passes through the point (-1, 2).

Write an equation for this line in point-slope form.

Answers

Answer:

Step-by-step explanation:

Slope = m = 3

(x₁ , y₁) = (-1 , 2)

Point slope form: y - y₁ = m(x - x₁)

y - 2 = 3(x - [-1] )

y - 2 = 3(x + 1)

y - 2 = 3*x + 3*1

y - 2 = 3x + 3

    y = 3x + 3 + 2

    y = 3x + 5

the red line below is perpendicular to which of the following

Answers

The red line is perpendicular to the y-axis.

Drag


the yellow point until an accurate "height" of the triangle is drawn. Afterwards, fill out the empty boxes below to determine the area of the triangle.

Answers

Answer:

I don't see a yellow point

Help!!!
Find the domain of the function. Write the answer in interval notation.

Answers

Answer:

A

Step-by-step explanation:

Given

f(x) = [tex]\frac{2}{3x+2}[/tex]

The denominator cannot be zero as this would make f(x) undefined.

Equating the denominator to zero and solving gives the value that x cannot be.

3x + 2 = 0 ⇒ 3x = - 2 ⇒ x = - [tex]\frac{2}{3}[/tex] ← excluded value

Then

domain is ( - ∞ , - [tex]\frac{2}{3}[/tex] ) U ( - [tex]\frac{2}{3}[/tex], ∞ ) → A

If f(x) = -3 3 and g(x)= 4x2 + 2x - 4, find (f +g)(x).

Answers

Answer:

A. 4x² + 9x/4 - 7

General Formulas and Concepts:

Pre-Algebra

Order of Operations: BPEMDAS

Brackets Parenthesis Exponents Multiplication Division Addition Subtraction Left to Right

Algebra I

Terms/CoefficientsFunctionsFunction Notation

Step-by-step explanation:

Step 1: Define

Identify

f(x) = x/4 - 3

g(x) = 4x² + 2x - 4

(f + g)(x) is f(x) + g(x)

Step 2: Find

Substitute in functions:                                                                                     (f + g)(x) = x/4 - 3 + 4x² + 2x - 4Combine like terms:                                                                                         (f + g)(x) = 4x² + 9x/4 - 7

These points are linear.
Find the slope.
x-3 -2 -1 0 1/2
y-3 -2 -1 0 1 2
slope = [?]

Answers

Answer:

Slope = 4

Step-by-step explanation:

The slope of a linear equation can be calculated as;

m = y2-y1/x2-x1

What we have to do here is to select any two points ;

(x1,y1) = (3,8)

(x2,y2) = (6,20)

m = (20-8)/(6-3) = 12/3 = 4

Find mCFB

Help me please

Answers

Answer:

A

Step-by-step explanation:

<CPA = 50o                     Given

<FPB = 50o                      Vertically opposite a 50 degree angle

<CPF = 90o                      Given

<CPB = <FPB + <CPF       Given or found

<CPB = 50 + 90                Substitute

<CPB = 140o

Match the base to the corresponding height.

Answers

Answer:

I can't see the picture

Step-by-step explanation:

SORRY :(

what do you understand by socialization​

Answers

Socialization is the lifelong process through which people learn the values and norms of a given society.

​When CP = Rs 1250 and SP = Rs 1500, find profit or loss percent ?

Answers

Answer:

profit=20%

Step-by-step explanation:

profit%=sp-cp/cp×100%

=1500-1250/1250×100

=250/1250×100

=20%

Solve for the questions (both of them) and label you answers for which question

Answers

Jjbbh I don’t under stand

PLS HELP ASAP, I need it in 10 mins. I GIVE 15 PTS !!!! if v1 = (3,-4) and v2 = (2,6) then v1*v2 is equal to which of the following?
A. 30
B. (-12, -24)
C. (6,-24)
D. -18

Answers

It is C. Good luck on ur test!!!!

v₁·v₂ is -18 which is correct option(D)

What are Arithmetic operations?

Arithmetic operations can also be specified by the subtract, divide, and multiply built-in functions.

The operator that perform arithmetic operation are called arithmetic operators .

Operators which let do basic mathematical calculations

+ Addition operation : Adds values on either side of the operator.

For example 4 + 2 = 6

- Subtraction operation : Subtracts right hand operand from left hand operand.

for example 4 -2 = 2

* Multiplication operation : Multiplies values on either side of the operator

For example 4*2 = 8

/ Division operation : Divides left hand operand by right hand operand

For example 4/2 = 2

Given that,

v1 = (3,-4) and v2 = (2,6)

To determine v₁·v₂

v₁·v₂ = 3·(2) - 4·(6)

v₁·v₂ = 6 - 24

v₁·v₂ = -18

Hence, the v₁·v₂ is -18.

Learn more about Arithmetic operations here:

brainly.com/question/25834626

#SPJ2

Ava’s gross pay is $2,500 per month. Her deductions include the following:

Federal income tax $290

State income tax $85

Social Security $112

Medicare $35

What is Ava’s net pay each month?

Answers

$1,978

Explanation:

Gross pay is what’s made before taxes and necessary deductions, net pay is after taxes and necessary deductions. So we minus the deductions to find the net pay. The equation is 2,500 - 290 - 85 - 112 - 35 = ? . The answer is $1,978.

I hope this helps. Please mark me the Brainliest, it’s not necessary but I put time and effort into every answer and I would appreciate it greatly. Have a great day, stay safe and stay healthy ! :)

Y=2/7 -7 y=-x+2 What is the solution for this system of equations?​

Answers

Answer:

(61/7, -47/7)

Step-by-step explanation:

Given: y=2/7 -7

           y=-x+2

Rewrite the top equation ( y=2/7 -7

y = -47/7

y = -x + 2

Because both equations are equal to y, we can rewrite it again.

-47/7 = -x + 2

Add x to both sides

x + (-47/7) = 2

Add 47/7 to both sides.

x = 47/7 + 2/1

x = 61/7

As stated in the beginning, y is equal to -47/7

Hope you understand!

If anyone can help with this ill mark Brainly

Answers

Answer:

In this case, we can do substitution.

Step-by-step explanation:

For the first one, (s - t)(x) = ((x - 5) - 4x^2)(x) = x^2 - 21x

For the second one, (s*t)(x) = ((x - 5) *4x^2)(x) = 4x^4 - 20x^3

And for the last one,  (s+t)(-2) = ((x - 5) + 4x^2)(-2) = -8x^2 - 2x + 10

Hope your happy with the answer :)

Shirley buys fiction books for $20 each, and then marks up by 25% to
resell. What is the markup in dollars?

Answers

Answer:

$5

Step-by-step explanation:

Find the markup by finding 25% of 20:

20(0.25)

= 5

So, the markup is $5

Please helpppp
Find h

Answers

Answer:

the square root of 85 (so 85 is the answer)

Step-by-step explanation:

You can use the pythagorean theorem to find the height. So the sides of the triangle would contain the radius (4/2 = 2) and 9.

so, 2 squared plus 9 squared = 85

I hope this helps!

CAN SOMEBODY PLEASE HELP MEEEE

Answers

Answer:

105.

Step-by-step explanation:

.

Graph the solution of the inequality 3/7(35x-14)<_ 21x/2+3

Answers

Answer:

You'll have a closed circle at x = 2, and shading to the left

See the diagram below

=========================================================

Explanation:

The fractions here are 3/7 and 21/2. The denominators of which are 7 and 2 respectively. The LCD is 7*2 = 14.

If we multiply both sides by 14, then this will clear out the denominators and make the fractions go away.

14*(3/7) = (14*3)/7 = 42/7 = 814*(21/2) = (14*21)/2 = 294/2 = 147

So if we multiplied both sides by 14, then we have these steps

[tex]\frac{3}{7}(35x-14) \le \frac{21x}{2}+3\\\\14*\frac{3}{7}(35x-14) \le 14*\left(\frac{21x}{2}+3\right)\\\\14*\frac{3}{7}(35x-14) \le 14*\left(\frac{21x}{2}\right)+14*\left(3\right)\\\\6(35x-14) \le 147x+42\\\\[/tex]

--------------------------

Let's isolate x

[tex]6(35x-14) \le 147x+42\\\\6(35x)+6(-14) \le 147x+42\\\\210x-84 \le 147x+42\\\\210x-147x \le 42+84\\\\63x \le 126\\\\x \le 126/63\\\\x \le 2\\\\[/tex]

The graph of this will consist of a closed or filled in circle at x = 2. We shade to the left to represent numbers smaller than 2.

So either x = 2 or x < 2.

If we used an open hole at 2, then we wouldn't be including 2 (but we want to include this endpoint).

See the diagram below.

Solve for x and y:
x – 3y = -8
3x + 2y = 31
Select one:
O a. (-11,-1)
O b. (11,1)
o c. (5,8)
O d. (7,5)

Answers

Answer:

O d. (7,5)

Step-by-step explanation:

x – 3y = -8 (1)

3x + 2y = 31 (2)

From (1)

x = -8 + 3y

Substitute x = -8 + 3y into (2)

3x + 2y = 31 (2)

3(-8 + 3y) + 2y = 31

-24 + 9y + 2y = 31

-24 + 11y = 31

11y = 31 + 24

11y = 55

y = 55/11

y = 5

Substitute y = 5 into (1)

x – 3y = -8 (1)

x - 3(5) = -8

x - 15 = -8

x = -8 + 15

x = 7

(x, y) = (7, 5)

URGENT PLS ANSWER QUICKLY​

Answers

Answer:

9th is 44 mark me as brainlist

Answer:

Question 9 is perimeter

If Wade has 2 times as many dimes as quarters and they have a combined value of 270 cents, how many of each coin does he have?

Answers

Answer:

Step-by-step explanation:

If he has twice the number of dimes as quarters, then obviously he has more dimes than quarters. The expression that represents that is

d = 2q

That relates the NUMBER of coins; now we need one that relates the VALUE which is a dollars and cents thing. We know that the combined value of the coins is $2.70. The expression that represents this is

.1d + .25q = 2.70 because dimes are worth .10 and quarters are worth .25

Subbing the first equation into the second gives us

.1(2q) + .25q = 2.70 and

.2q + .25q = 2.70 and

.45q = 2.70 so

q = 6

This means he has 6 quarters. If the umber of dimes is twice as much, then d = 2(6) and d = 12.

He has 6 quarters and 12 dimes

Can someone help with problems fivethroughseven

Answers

Answer:

5) 15120

6) 11880

7) 336

Step-by-step explanation:

The formula for permutation where mPn is m!/(m-n)!

Applying this to question 5, we get 9!/4!, which is 15120.

For question 6, we get 12!/8!, which is 11880.

For question 7, we get 8!/5!, which is 336.

Which of the following is most likely the next step in the series?
А.
B.
C.
D.

Answers

Answer:

D

Step-by-step explanation:

The answer is D because the pattern adds one angle every one shape. Ex. Triangle, Rectangle, Pentagon, Hexagon

Which is the area between the x-axis and y=x from x=1 to x=5

Answers

Answer:

[tex]\displaystyle A = 12[/tex]

General Formulas and Concepts:

Pre-Algebra

Order of Operations: BPEMDAS

BracketsParenthesisExponentsMultiplicationDivisionAdditionSubtractionLeft to Right

Algebra I

FunctionsFunction NotationGraphing

Calculus

Integrals

Definite IntegralsArea under the curve

Integration Rule [Reverse Power Rule]:                                                               [tex]\displaystyle \int {x^n} \, dx = \frac{x^{n + 1}}{n + 1} + C[/tex]

Integration Rule [Fundamental Theorem of Calculus 1]:                                     [tex]\displaystyle \int\limits^b_a {f(x)} \, dx = F(b) - F(a)[/tex]

Area of a Region Formula:                                                                                     [tex]\displaystyle A = \int\limits^b_a {[f(x) - g(x)]} \, dx[/tex]

Step-by-step explanation:

Step 1: Define

Identify

y = x

Interval: x = 1 to x = 5

Step 2: Sort

Graph the function. See Attachment.

Bounds of Integration: [1, 5]

Step 3: Find Area

Substitute in variables [Area of a Region Formula]:                                   [tex]\displaystyle A = \int\limits^5_1 {x} \, dx[/tex][Integral] Integrate [Integration Rule - Reverse Power Rule]:                     [tex]\displaystyle A = \frac{x^2}{2} \bigg| \limits^5_1[/tex]Evaluate [Integration Rule - Fundamental Theorem of Calculus 1]:           [tex]\displaystyle A = 12[/tex]

Topic: AP Calculus AB/BC (Calculus I/I + II)

Unit: Integration

Book: College Calculus 10e

Find the equation of a line that is perpendicular to y = -3x – 1 and passes through the point
(3,2).
Give your answer in the form y = mx + b.

Answers

the answer is:
y = 1/3x + 1
I think the answer is y= ⅓ + 1

PLEASE HELP ASAP WHAT IS THE ANSWER TO THIS QUADRATIC EQUATIONNNN

Answers

Answer:

pls i don't know

The answer is A. 2

This is because the stretch of the parabola is elongated by 2!

Find the value of x from the following given figures.​

Answers

solution :-

here,

We know that interior opposite angles are equal.

So,

110° = 50° + x (being interior opposite angles)

110° - 50° = x

60° = x

the value of x =60°

hope it is helpful to you ☺️

Other Questions
"Infant safe haven laws" provide all of the following EXCEPTA. protection to unwanted babies from being abandoned or endangered B. protection to the parent against criminal prosecution C. protection to the parent to remain anonymous D. protection to the parent to later reclaim the child PreguntasIn Spanish, write logical questions that would elicit the responses given below.changed1. Empec a ir al gimnasio el mes pasado.2.La vi ayer.3.Aqulla es amarilla.4.Son doce dlares.5.Diego me prest su motocicleta. Do you think it's possible for people to travel to the center of the planet? Why or why not? Quantity (Units) Private Value (Dollars) Private Cost (Dollars) External Cost (Dollars) 1 46 21 6 2 44 24 6 3 42 27 6 4 40 30 6 5 38 33 6 6 36 36 6 7 34 39 6The table represents a market in which: a. there is no externalityb. there is a positive externality. c. there is a negative externality. d. The answer cannot be determined from inspection of the table. Can someone help me with this? Thanks! write a function rule for the following data hello! I'm having a bit of trouble trying to solve this, could anyone help explain it to me? What is the range of the given set of ordered pairs?(9,-2) (4,3) ( 8, 10) (-4, 8) bakit itinuring angkop na angkop ang mga nilalamab ng ibong adarna sa kalinangan at kulturang pilipino Assume the following information: Selling price per unit $200 Contribution margin ratio 50% Total fixed costs $275,000 How many units must be sold to generate a profit of $50,000 The center of the circle is located (3'8), and the circle has a radius that is 5 units long. What is the general form of the equation for the circle Based on this model, households earn income when firms __________ purchase goods and services ___________in markets for factors of production. Suppose Caroline earns $625 per week working as jewelry appraiser for Classy's Jewelry Store. She uses $10 to get her car washed at Spotless Car Wash. Spotless Car Wash pays Antonio $275 per week to wash cars. Antonio uses $150 to purchase a necklace from Classy's Jewelry Store. Identify whether each of the following events in this scenario occurs in the market for factors of production or the market for goods and services. Event Market for Factors of Production Market for Goods and Services Caroline spends $10 to get her car washed. Antonio spends $150 to purchase a necklace from Classy's Jewelry Store. Antonio earns $275 per week working for Spotless Car Wash. Which of the elements of this scenario represent a flow from a household to a firm? This could be a flow of dollars, inputs, or outputs. i. The car wash Caroline receives ii. The $275 per week iii. Antonio earns working for Spotless Car wash The $150 Antonio spends to purchase a necklace from Classy's Jewelry Store The actual economy is more complicated than the one illustrated in the previous circular-flow diagram of a simple economy True / False Please help NO LINKSUse cylindrical shells to find the volume of the solid obtained by rotating the region bounded by y=x2 , y=0 , and x=5, about the y -axis.V= What is the vertex of the graph of the function f(x) = (x + 2)^2 3? According to Piaget, middle childhood's advances of concrete operations include new abilities for performing tasks of __________. Z Is An Even Integar Greater Than 30 And Less Than Or Equal To 34. Find the missing angle measures. Jansen Company reports the following for its ski department for the year 2019. All of its costs are direct, except as noted. Sales $ 605,000 Cost of goods sold 425,000 Salaries 115,000 ($25,200 is indirect) Utilities 14,500 ($5,800 is indirect) Depreciation 48,600 ($17,500 is indirect) Office expenses 28,200 (all indirect) 1. Prepare a departmental income statement for 2019. 2. What are emotive action or doing words?A. Active voiceB. ImperativeC. Powerful verbD. Passive voice One region on Earth that has become a rich source of new meteorites in recent decades (including the meteorite from Mars that got famous because some scientists claimed they had found evidence for the building blocks of life on Mars) is: